How Does the Chain Rule Apply in Polar Coordinates?

In summary, using the chain rule, it can be shown that ## \frac{1}{r^{2}}\frac{\partial ^{2} z}{\partial \theta ^{2}} = sin^{2}(\theta)\frac{\partial ^{2} z}{\partial x^{2}}-2sin(\theta)cos(\theta)\frac{\partial ^{2} z}{\partial x \partial y}+cos^{2}(\theta)\frac{\partial ^{2} z}{\partial y ^{2}} - \frac{1}{r}\frac{\partial z}{\partial r}##.
  • #1
JohnTheMan
3
0

Homework Statement


z = ƒ(x,y), x = rcos(θ), y = rsin(θ)
Use the chain rule to show that:
[tex] \frac{1}{r^{2}}\frac{\partial ^{2} z}{\partial \theta ^{2}} = sin^{2}(\theta)\frac{\partial ^{2} z}{\partial x^{2}}-2sin(\theta)cos(\theta)\frac{\partial ^{2} z}{\partial x \partial y}+cos^{2}(\theta)\frac{\partial ^{2} z}{\partial y ^{2}} - \frac{1}{r}\frac{\partial z}{\partial r} [/tex]

Homework Equations


∂z/∂r = (∂z/∂x)(∂x/∂r) + (∂z/∂y)(∂y/∂r)

The Attempt at a Solution


I have worked it through and I have found that:
[tex] \frac{\partial z}{\partial \theta } = -rsin(\theta)\frac{\partial z}{\partial x}+rcos(\theta)\frac{\partial z}{\partial y}[/tex]
[tex] \frac{\partial z}{\partial r } = cos(\theta)\frac{\partial z}{\partial x}+sin(\theta)\frac{\partial z}{\partial y}[/tex]
[tex] \frac{\partial ^{2} z}{\partial \theta ^{2}} = r^{2}(sin^{2}(\theta)\frac{\partial ^{2} z}{\partial x^{2}}-2sin(\theta)cos(\theta)\frac{\partial ^{2} z}{\partial x \partial y}+cos^{2}(\theta)\frac{\partial ^{2} z}{\partial y ^{2}}) [/tex]
I have checked these a number of times and they seem right to me. The last one listed is the second partial derivative of z in terms of θ, but it doesn't match that of the problem statement. I'm not really sure what I'm doing wrong. Thanks :)
 
Physics news on Phys.org
  • #2
Your second derivative looks like you just squared your first derivative.
You need to actually take one more derivative to get ## \frac{\partial^2 z}{\partial \theta^2}##.
 
  • #3
RUber said:
Your second derivative looks like you just squared your first derivative.
You need to actually take one more derivative to get ## \frac{\partial^2 z}{\partial \theta^2}##.
I didn't square it, I took the second derivative.
 
  • #4
You have the first part right. But your second derivative is wrong.
##\frac{\partial}{\partial \theta } f(x,y) = \frac{\partial f}{\partial x} \frac{\partial x}{\partial \theta} + \frac{\partial f}{\partial y} \frac{\partial y}{\partial \theta}##
##\frac{\partial}{\partial \theta }\left( \frac{\partial}{\partial \theta } f(x,y)\right) = \frac{\partial}{\partial \theta }\left( \frac{\partial f}{\partial x} \frac{\partial x}{\partial \theta} + \frac{\partial f}{\partial y} \frac{\partial y}{\partial \theta}\right) ##
Which is
## = \left( \frac{\partial}{\partial \theta }\frac{\partial f}{\partial x}\right) \frac{\partial x}{\partial \theta}+\underline{ \frac{\partial f}{\partial x} \left( \frac{\partial}{\partial \theta }\frac{\partial x}{\partial \theta} \right) } + \left( \frac{\partial}{\partial \theta } \frac{\partial f}{\partial y}\right) \frac{\partial y}{\partial \theta}+ \underline{ \frac{\partial f}{\partial y} \left( \frac{\partial}{\partial \theta }\frac{\partial y}{\partial \theta}\right)} ##
In your expansion, you missed the second derivatives with respect to theta on x and y. Which will give you the equivalent parts you need for the partial with respect to r.
 
  • Like
Likes JohnTheMan
  • #5
RUber said:
You have the first part right. But your second derivative is wrong.
##\frac{\partial}{\partial \theta } f(x,y) = \frac{\partial f}{\partial x} \frac{\partial x}{\partial \theta} + \frac{\partial f}{\partial y} \frac{\partial y}{\partial \theta}##
##\frac{\partial}{\partial \theta }\left( \frac{\partial}{\partial \theta } f(x,y)\right) = \frac{\partial}{\partial \theta }\left( \frac{\partial f}{\partial x} \frac{\partial x}{\partial \theta} + \frac{\partial f}{\partial y} \frac{\partial y}{\partial \theta}\right) ##
Which is
## = \left( \frac{\partial}{\partial \theta }\frac{\partial f}{\partial x}\right) \frac{\partial x}{\partial \theta}+\underline{ \frac{\partial f}{\partial x} \left( \frac{\partial}{\partial \theta }\frac{\partial x}{\partial \theta} \right) } + \left( \frac{\partial}{\partial \theta } \frac{\partial f}{\partial y}\right) \frac{\partial y}{\partial \theta}+ \underline{ \frac{\partial f}{\partial y} \left( \frac{\partial}{\partial \theta }\frac{\partial y}{\partial \theta}\right)} ##
In your expansion, you missed the second derivatives with respect to theta on x and y. Which will give you the equivalent parts you need for the partial with respect to r.
RUber said:
You have the first part right. But your second derivative is wrong.
##\frac{\partial}{\partial \theta } f(x,y) = \frac{\partial f}{\partial x} \frac{\partial x}{\partial \theta} + \frac{\partial f}{\partial y} \frac{\partial y}{\partial \theta}##
##\frac{\partial}{\partial \theta }\left( \frac{\partial}{\partial \theta } f(x,y)\right) = \frac{\partial}{\partial \theta }\left( \frac{\partial f}{\partial x} \frac{\partial x}{\partial \theta} + \frac{\partial f}{\partial y} \frac{\partial y}{\partial \theta}\right) ##
Which is
## = \left( \frac{\partial}{\partial \theta }\frac{\partial f}{\partial x}\right) \frac{\partial x}{\partial \theta}+\underline{ \frac{\partial f}{\partial x} \left( \frac{\partial}{\partial \theta }\frac{\partial x}{\partial \theta} \right) } + \left( \frac{\partial}{\partial \theta } \frac{\partial f}{\partial y}\right) \frac{\partial y}{\partial \theta}+ \underline{ \frac{\partial f}{\partial y} \left( \frac{\partial}{\partial \theta }\frac{\partial y}{\partial \theta}\right)} ##
In your expansion, you missed the second derivatives with respect to theta on x and y. Which will give you the equivalent parts you need for the partial with respect to r.
Ahh. That makes sense. Thanks a lot.
 

Related to How Does the Chain Rule Apply in Polar Coordinates?

1. How do I use the chain rule to solve a problem?

The chain rule is a calculus technique used to find the derivative of a composite function. To use it, you need to first identify the inner and outer functions of the composite function. Then, take the derivative of the outer function and multiply it by the derivative of the inner function. This will give you the derivative of the composite function.

2. Can you provide an example of using the chain rule?

Yes, for example, if you have the function f(x) = (x^2 + 1)^3, the inner function would be (x^2 + 1) and the outer function would be (x)^3. To find the derivative, you would first take the derivative of the outer function, which would be 3x^2. Then, you would multiply it by the derivative of the inner function, which would be 2x. This gives you the derivative of the composite function, which is 6x(x^2 + 1)^2.

3. What is the purpose of using the chain rule?

The chain rule is used to find the derivative of a composite function, where two or more functions are combined together. It allows us to break down a complex function into smaller parts and find the rate of change at a specific point. This tool is especially useful in physics, engineering, and other fields that require the use of calculus.

4. Are there any common mistakes to avoid when using the chain rule?

Yes, some common mistakes to avoid when using the chain rule include not identifying the inner and outer functions correctly, forgetting to multiply by the derivative of the inner function, and making algebraic errors when simplifying the final answer. It is important to carefully follow the steps of the chain rule and double check your work to avoid these mistakes.

5. Can the chain rule be applied to other types of functions besides composite functions?

Yes, the chain rule can also be applied to other types of functions such as implicit functions, inverse functions, and logarithmic functions. However, the process may vary slightly depending on the type of function. It is important to understand the concept of the chain rule and how to apply it to different types of functions in order to solve problems effectively.

Similar threads

  • Calculus and Beyond Homework Help
Replies
3
Views
2K
  • Calculus and Beyond Homework Help
Replies
8
Views
1K
  • Calculus and Beyond Homework Help
Replies
1
Views
465
  • Calculus and Beyond Homework Help
Replies
3
Views
1K
  • Calculus and Beyond Homework Help
Replies
3
Views
674
  • Calculus and Beyond Homework Help
Replies
3
Views
855
  • Calculus and Beyond Homework Help
Replies
2
Views
819
  • Calculus and Beyond Homework Help
Replies
4
Views
1K
  • Calculus and Beyond Homework Help
Replies
18
Views
2K
  • Calculus and Beyond Homework Help
Replies
6
Views
1K
Back
Top